As I said in previous post the formal definition of the Riemann Integral is very useful when solving Olympiad problems.
Problem 1. Find the following limit
n→∞lim(n+11+n+21+...+2n1)
Solution. Let try to make our sum into something more or less similar to Riemann sum.
i=1∑nn+i1=n1(i=1∑n1+ni1).
Does this remind you of the monstrous i=1∑nf(ξi)(xi−xi−1)? But what if set xi=ni and consider the right Riemann sum? Now it will transform into n1(i=1∑nf(ni)).
From the last formula we can easily understand what function f we need to consider, so by the definition:
n→∞lim(n+11+n+21+...+2n1)=n1(i=1∑n1+ni1)=
=n1(i=1∑nf(ni))=∫01x+11=ln2.
Now using the same approach try solving the following problems.
Problem 2. n→∞lim(n+11+n+21+...+4n1)
Problem 3. n→∞lim(n2+1n+n2+4n+...+n2+n2n)
#Calculus
#Limits
#Integration
#Goldbach'sConjurersGroup
Easy Math Editor
This discussion board is a place to discuss our Daily Challenges and the math and science related to those challenges. Explanations are more than just a solution — they should explain the steps and thinking strategies that you used to obtain the solution. Comments should further the discussion of math and science.
When posting on Brilliant:
*italics*
or_italics_
**bold**
or__bold__
paragraph 1
paragraph 2
[example link](https://brilliant.org)
> This is a quote
\(
...\)
or\[
...\]
to ensure proper formatting.2 \times 3
2^{34}
a_{i-1}
\frac{2}{3}
\sqrt{2}
\sum_{i=1}^3
\sin \theta
\boxed{123}
Comments
Problem 2 is same as Problem 1, the only difference is that the integration limits change. The lower limit is 0 and upper limit is 3. Hence, the answer is ln4.
Problem 3 can be written as:
n→∞limn1(r=1∑n1+(nr)21)
The above is equivalent to
∫011+x2dx=4π
Log in to reply
Good job!
bt i dnt undrstand how it is 1 + x^2 & in the problem 1 only x??
Log in to reply
Because to get the answer we have to consider a variable (say x) Which will be in form of r/n after modifying the problem.
Is this from IMC ?
Log in to reply
Sorry but I didn't get your question. Can you please explain what is "IMC"?
Log in to reply
in problem 3. how we can put the limits from 0 to 3??????????